Đến nội dung

LilTee

LilTee

Đăng ký: 30-10-2011
Offline Đăng nhập: 16-01-2019 - 21:59
*****

#542256 Bất đẳng thức qua các đề thi HSG môn Toán của các trường, các tỉnh trên cả nư...

Gửi bởi LilTee trong 29-01-2015 - 16:44

Gửi tặng mọi người tài liệu "Bất đẳng thức qua các đề thi HSG môn Toán của các trường, các tỉnh trên cả nước 2014 – 2015 - Tăng Hải Tuân"
Trong quá trình làm không tránh khỏi sai sót, rất mong nhận được ý kiến đóng góp của các bạn để tài liệu hoàn thiện hơn.
Thân,
Tăng Hải Tuân  




#540620 VMO 2015

Gửi bởi LilTee trong 13-01-2015 - 04:01

5 cách giải câu Bất đẳng thức




#398491 $\sum \frac{1}{a\sqrt{a+b}}...

Gửi bởi LilTee trong 20-02-2013 - 02:30

Cho a,b,c là các số thực dương. Chứng minh rằng:
$\sum \frac{1}{a\sqrt{a+b}}\geq \frac{3}{\sqrt{2abc}}$


Bất đẳng thức cần chứng minh tương đương $$\frac{bc}{\sqrt{abc(a+b)}}+\frac{ca}{\sqrt{abc(b+c)}}+\frac{ab}{\sqrt{abc(c+a)}}\ge \frac{3}{\sqrt{2}}$$
Để cho dễ nhìn, ta đặt $x=ab,y=bc,z=ca$. Khi đó chúng ta cần chứng minh $$\frac{y}{\sqrt{2x(y+z)}}+\frac{z}{\sqrt{2y(x+z)}}+\frac{x}{\sqrt{2z(y+x)}}\ge \frac{3}{2}$$
Sử dụng bất đẳng thức $AM-GM$ ta có $$\frac{y}{\sqrt{2x(y+z)}}+\frac{z}{\sqrt{2y(x+z)}}+\frac{x}{\sqrt{2z(x+y)}}\ge \frac{2y}{2x+y+z}+\frac{2z}{2y+x+z}+\frac{2x}{2z+x+y}$$ Bây giờ, sử dụng bất đẳng thức $Cauchy-Schwarz$ và bất đẳng thức quen thuộc $xy+yz+zx\le \dfrac{{{\left( x+y+z \right)}^{2}}}{3}$ ta thấy rằng $$\begin{align}
\frac{2y}{2x+y+z}+\frac{2z}{2y+x+z}+\frac{2x}{2z+x+y}&=2\left( \frac{{{y}^{2}}}{2xy+{{y}^{2}}+yz}+\frac{{{z}^{2}}}{2yz+xz+{{z}^{2}}}+\frac{{{x}^{2}}}{2zx+{{x}^{2}}+yx} \right) \\
& \ge \frac{2{{\left( x+y+z \right)}^{2}}}{{{(x+y+z)}^{2}}+xy+yz+zx} \\
& \ge \frac{2{{\left( x+y+z \right)}^{2}}}{{{(x+y+z)}^{2}}+\dfrac{{{\left( x+y+z \right)}^{2}}}{3}} \\
& =\frac{3}{2}. \\
\end{align}$$ Bất đẳng thức được chứng minh. :)
Đẳng thức xảy ra khi và chỉ khi $a=b=c.$

Một kết quả tương tự:
Chứng minh rằng với mọi số thực dương $a, b, c$ ta luôn có
$$\frac{1}{a\sqrt{a+2b}}+\frac{1}{b\sqrt{b+2c}}+ \frac{1}{c\sqrt{c+2a}}\ge \sqrt{\frac{3}{abc}} $$


#339474 $$\sum \dfrac{1}{a(b+1)}\ge...

Gửi bởi LilTee trong 24-07-2012 - 01:39

Bài toán
Cho $a,b,c$ là các số thực dương. Chứng minh rằng :
$$\dfrac{1}{a(b+1)}+\dfrac{1}{b(c+1)}+\dfrac{1}{c(a+1)}\ge\dfrac{3}{\sqrt[3]{abc}\left (1+\sqrt[3]{abc}\right )}$$

\[\begin{align}
& \bullet a=k\frac{x}{y},b=k\frac{y}{z},c=k\frac{z}{x}\to \sqrt[3]{abc}=k \\
& \bullet \sum\limits_{cyc}{\frac{1}{a(b+1)}}=\sum\limits_{cyc}{\frac{1}{k\frac{x}{y}\left( k\frac{y}{z}+1 \right)}}=\frac{1}{k}\sum\limits_{cyc}{\frac{yz}{kxy+xz}}\ge \frac{1}{k}.\frac{{{\left( xy+yz+zx \right)}^{2}}}{\left( k+1 \right)xyz\left( x+y+z \right)}\ge \frac{3}{k\left( k+1 \right)}. \\
\end{align}\] Kết thúc chứng minh. $\blacksquare$


#337464 $$\sqrt{\dfrac{a(b+c)}{a^2+bc}...

Gửi bởi LilTee trong 19-07-2012 - 01:09

Bài toán [3]
Cho các số thực không âm $a,b,c$. Chứng minh rằng :
$$\sqrt{\dfrac{a(b+c)}{a^2+bc}}+\sqrt{\dfrac{b(c+a)}{b^2+ca}}+\sqrt{\dfrac{c(a+b)}{c^2+ab}}\ge 2$$

Dễ thấy đẳng thức xảy ra khi một biến bằng không, hai biến còn lại bằng nhau.
Trước hết, mục tiêu của ta là phá căn, hiển nhiên tư tưởng đầu tiên sẽ sử dụng $AM-GM.$ Ta sẽ đánh giá sao cho đẳng thức đạt tại $a=b>0$ và $c=0$ chẳng hạn.
Chú ý với đẳng thức trên thì $a(b+c)=a^2+bc, \ b(c+a)=b^2+ca$ và ta sẽ sử dụng $AM-GM.$
$$\begin{align}
& \sqrt{\frac{a\left( b+c \right)}{{{a}^{2}}+bc}}=\frac{2a\left( b+c \right)}{2\sqrt{a\left( b+c \right)\left( {{a}^{2}}+bc \right)}}\ge \frac{2a\left( b+c \right)}{a\left( b+c \right)+\left( {{a}^{2}}+bc \right)}=\frac{2a\left( b+c \right)}{\left( a+c \right)\left( a+b \right)} \\
& \sqrt{\frac{b(c+a)}{{{b}^{2}}+ca}}=\frac{2b(c+a)}{2\sqrt{b(c+a)\left( {{b}^{2}}+ca \right)}}\ge \frac{2b(c+a)}{b(c+a)+\left( {{b}^{2}}+ca \right)}=\frac{2b(c+a)}{\left( a+b \right)\left( c+b \right)} \\
\end{align}$$
Như vậy còn đại lượng $c(a+b)$ và $c^2+ab$ thì sao? Hiển nhiên khi $a=b, \ c=0$ thì hai đại lượng đó không thể bằng nhau, liệu ta có thể sử dụng $AM-GM$ giống như trên được không? Câu trả lời là có! Chú ý rằng $\sqrt{\dfrac{c(a+b)}{c^2+ab}}$ có $c$ ở trên tử, nên nếu ta giữ nguyên $c$ ở trên tử, thì mọi đánh giá của ta đối với $\sqrt{\dfrac{c(a+b)}{c^2+ab}}$ mà không làm mất $c$, thì ta luôn giữ được đẳng thức tại $c=0$. Nhưng ta không thể viết $$\sqrt{\frac{c(a+b)}{{{c}^{2}}+ab}}=\frac{2c\left( a+b \right)}{2\sqrt{c\left( a+b \right)\left( {{c}^{2}}+ab \right)}}$$ để sử dụng $AM-GM$ ở dưới mẫu vì viết như thế, khi $c=0$ thì phân thức sẽ vô nghĩa ngay. Vậy giải quyết như thế nào? Rất đơn giản, ta trình bày như sau:
Ta sẽ chứng minh $$\sqrt{\frac{c(a+b)}{{{c}^{2}}+ab}}\ge \frac{2c\left( a+b \right)}{c\left( a+b \right)+\left( {{c}^{2}}+ab \right)}=\frac{2c\left( a+b \right)}{\left( c+a \right)\left( c+b \right)},$$ tương đương
$$\begin{matrix}
\frac{c(a+b)}{{{c}^{2}}+ab}\ge {{\left[ \frac{2c\left( a+b \right)}{c\left( a+b \right)+\left( {{c}^{2}}+ab \right)} \right]}^{2}}, \\
c\left( a+b \right){{\left[ c\left( a+b \right)+\left( {{c}^{2}}+ab \right) \right]}^{2}}\ge c\left( a+b \right).4c\left( a+b \right)\left( {{c}^{2}}+ab \right), \\
c\left( a+b \right){{\left[ c\left( a+b \right)-\left( {{c}^{2}}+ab \right) \right]}^{2}}\ge 0. \\
\end{matrix}$$
Vậy bất đẳng thức trên đúng và vẫn đảm bảo đẳng thức tại $c=0$. Từ đó ta có
\[\begin{align}
\sqrt{\frac{a(b+c)}{{{a}^{2}}+bc}}+\sqrt{\frac{b(c+a)}{{{b}^{2}}+ca}}+\sqrt{\frac{c(a+b)}{{{c}^{2}}+ab}} & \ge \frac{2\left[ a{{\left( b+c \right)}^{2}}+b{{\left( c+a \right)}^{2}}+c{{\left( a+b \right)}^{2}} \right]}{\left( a+b \right)\left( b+c \right)\left( c+a \right)} \\
& =2\left[ 1+\frac{4abc}{\left( a+b \right)\left( b+c \right)\left( c+a \right)} \right] \\
& \ge 2. \\
\end{align}\] Kết thúc chứng minh.
Đẳng thức xảy ra khi $a=b, \ c=0$ hoặc các hoán vị. $\blacksquare$


#334007 [TOPIC] Nhiều cách giải cho một bài toán. Bạn chọn cách nào?

Gửi bởi LilTee trong 10-07-2012 - 15:05

Sử dụng bất đẳng thức $AM-GM$, ta có : $$(ab+bc+ca)\left (\dfrac{a}{b^2+9}+\dfrac{b}{c^2+9}+\dfrac{c}{a^2+9}\right )\le \dfrac{1}{2}(ab+bc+ca)\left (\dfrac{a}{b+4}+\dfrac{b}{c+4}+\dfrac{c}{a+4}\right )$$ Do đó bài toán sẽ được chứng minh nếu ta chỉ ra được
$$(ab+bc+ca)\left (\dfrac{a}{b+4}+\dfrac{b}{c+4}+\dfrac{c}{a+4}\right )\le \dfrac{9}{5}$$ hay tương đương
$$\dfrac{(ab+bc+ca)\left [ab^2+bc^2+ca^2+84-4(ab+bc+ca)\right ]}{abc+4(ab+bc+ca)+112}\le \dfrac{9}{5}$$ Sử dụng bổ đề quen thuộc $$(ab+bc+ca)(ab^2+bc^2+ca^2) \le 9,$$ ta đưa bài toán về chứng minh $$9\left( r+4q+112 \right)\ge 5\left( 9+84q-4{{q}^{2}} \right)$$ hay tương đương $$9r+20{{q}^{2}}-384q+963\ge 0.$$ Theo $Schur$, ta có $9r \ge 3(4q-9)$, do đó $$\begin{align}
9r+20{{q}^{2}}-384q+963 & \ge 3\left( 4q-9 \right)+20{{q}^{2}}-384q+963 \\
& =4\left( q-3 \right)\left( 5q-78 \right) \\
& \ge 0. \\
\end{align}$$ Có được đánh giá cuối vì $q\le3$. Bài toán được chứng minh xong ;).
Đẳng thức xảy ra khi và chỉ khi $a=b=c=1. \ \blacksquare$


#333982 Topic bất đẳng thức THCS (2)

Gửi bởi LilTee trong 10-07-2012 - 13:37

Bài 439: Cho $a,b,c$ là các số thực dương thoả mãn $a^3+b^3+c^3=3$. Chứng minh rằng
$$\frac{ab}{\sqrt[3]{{{b}^{3}}+8}}+\frac{bc}{\sqrt[3]{{{c}^{3}}+8}}+\frac{ca}{\sqrt[3]{{{a}^{3}}+8}}\le \frac{\sqrt{{{a}^{2}}+{{b}^{2}}+{{c}^{2}}}}{\sqrt[6]{3}}$$
Bài 440: Cho $a,b,c$ là các số thực dương thoả mãn $a^2+ b^2+ c^2=3$. Chứng minh rằng $$\frac{ab}{\sqrt[3]{b+2}}+\frac{bc}{\sqrt[3]{{c+2}}}+\frac{ca}{\sqrt[3]{{a+2}}}\le \sqrt[3]{9}$$
Bài 441: Cho $a,b,c$ là các số thực dương thoả mãn $a^2+ b^2+ c^2=3$. Hãy chứng minh rằng
$$\frac{ac}{\sqrt[3]{b+2}}+\frac{ba}{\sqrt[3]{{c+2}}}+\frac{cb}{\sqrt[3]{{a+2}}}\le \sqrt[3]{3(a+b+c)}$$
PS: Mấy bài này na ná nhau thôi :)


#332788 Topic bất đẳng thức THCS (2)

Gửi bởi LilTee trong 07-07-2012 - 11:10

Hic, hình như mọi người post bài hơi quá đà rồi ? Topic Bất đẳng thức THCS mà lôi cả VMO vào làm, rồi có cả bất đẳng thức 4 biến nữa ? :wacko: . Thế này thì các em THCS còn gì dám làm bất đẳng thức nữa? :angry:
Mấy bài mới post, đưa cho mấy anh chị THPT cũng chưa chắc đã làm được, nếu không đam mê bất đẳng thức, nói gì đến các em THCS :wacko: .
Mình có đôi điều góp ý vậy thôi.
Lil.Tee


#332656 $4(xy+yz+zx)\leq \sqrt{(x+y)(y+z)(z+x)}(\sqrt{x+y}+\...

Gửi bởi LilTee trong 06-07-2012 - 21:22

Cách khác :mellow:
Chuẩn hóa ab+bc+ca=3 $\Rightarrow a+b+c\geq 3,abc\leq 1$
Ta có : $(a+b)(b+c)(c+a)= (a+b+c)(ab+bc+ca)-abc\geq 3.3-1= 8$
$VT\geq 3\sqrt[6]{(x+y)(y+z)(z+x)}.\sqrt{(x+y)(y+z)(z+x)}\geq 12= 4(ab+bc+ca)$

Cách của bạn và cách của mình thực ra chỉ là một thôi :).


#332561 $4(xy+yz+zx)\leq \sqrt{(x+y)(y+z)(z+x)}(\sqrt{x+y}+\...

Gửi bởi LilTee trong 06-07-2012 - 16:24

Bài toán:
Ch0 $x,y,z>0$ chứng minh rằng:
$$4(xy+yz+zx)\leq \sqrt{(x+y)(y+z)(z+x)}(\sqrt{x+y}+\sqrt{y+z}+\sqrt{x+z})$$

Sử dụng trực tiếp bất đẳng thức AM-GM bộ ba số, ta đưa bài toán về chứng minh bất đẳng thức mạnh hơn sau: $$\begin{align}
& 3\sqrt[6]{(x+y)(y+z)(z+x)}.\sqrt{(x+y)(y+z)(z+x)}\ge 4(xy+yz+zx) \\
& \Leftrightarrow {{3}^{6}}{{(x+y)}^{4}}{{(y+z)}^{4}}{{(z+x)}^{4}}\ge {{4}^{6}}{{(xy+yz+zx)}^{6}} \\
\end{align}$$ Sử dụng bất đẳng thức $$9(x+y)(y+z)(z+x) \ge 8(x+y+z)(xy+yz+zx)$$ ta đưa về chứng minh $(x+y+z)^2 \ge 3(xy+yz+zx).$
Hiển nhiên đúng.


#332556 Chứng minh: $\sqrt {a + {{(b - c)}^2}} + \sqrt {b + {{(c - a)...

Gửi bởi LilTee trong 06-07-2012 - 16:06

Bài toán: Cho $a,b,c$ là các số không âm thỏa mãn $a+b+c=1$. Chứng minh:
\[\sqrt {a + {{(b - c)}^2}} + \sqrt {b + {{(c - a)}^2}} + \sqrt {c + {{(a - b)}^2}} \ge \sqrt{3}\]

Không mất tính tổng quát, giả sử $a\ge b\ge c.$
Sử dụng trực tiếp bất đẳng thức véc-tơ, ta có \[\begin{align}
\sqrt{a+{{(b-c)}^{2}}}+\sqrt{b+{{(c-a)}^{2}}}+\sqrt{c+{{(a-b)}^{2}}}&=\sqrt{a+{{(b-c)}^{2}}}+\sqrt{b+{{(a-c)}^{2}}}+\sqrt{c+{{(a-b)}^{2}}} \\
& \ge \sqrt{{{\left( \sqrt{a}+\sqrt{b}+\sqrt{c} \right)}^{2}}+{{\left[ (b-c)+(a-c)+(a-b) \right]}^{2}}}. \\
& =\sqrt{{{\left( \sqrt{a}+\sqrt{b}+\sqrt{c} \right)}^{2}}+4{{\left( a-c \right)}^{2}}}. \\
\end{align}\] Như vậy, bài toán được chứng minh nếu ta chỉ ra được $${{\left( \sqrt{a}+\sqrt{b}+\sqrt{c} \right)}^{2}}+4{{\left( a-c \right)}^{2}}\ge 3\left( a+b+c \right),$$ hay tương đương $$4{{\left( a-c \right)}^{2}}\ge {{\left( \sqrt{a}-\sqrt{b} \right)}^{2}}+{{\left( \sqrt{b}-\sqrt{c} \right)}^{2}}+{{\left( \sqrt{c}-\sqrt{a} \right)}^{2}}.$$ Ta có $$\begin{align}
{{\left( \sqrt{a}-\sqrt{b} \right)}^{2}}+{{\left( \sqrt{b}-\sqrt{c} \right)}^{2}}+{{\left( \sqrt{a}-\sqrt{c} \right)}^{2}} & \le {{\left[ \left( \sqrt{a}-\sqrt{b} \right)+\left( \sqrt{b}-\sqrt{c} \right) \right]}^{2}}+{{\left( \sqrt{a}-\sqrt{c} \right)}^{2}} \\
& =2{{\left( \sqrt{a}-\sqrt{c} \right)}^{2}}. \\
\end{align}$$ Nên công việc của ta chỉ còn phải chứng minh \[\begin{align}
& 2{{\left( a-c \right)}^{2}}\ge {{\left( \sqrt{a}-\sqrt{c} \right)}^{2}} \\
& \Leftrightarrow 2{{\left( \sqrt{a}-\sqrt{c} \right)}^{2}}{{\left( \sqrt{a}+\sqrt{c} \right)}^{2}}\ge {{\left( \sqrt{a}-\sqrt{c} \right)}^{2}} \\
& \Leftrightarrow {{\left( \sqrt{a}-\sqrt{c} \right)}^{2}}\left[ 2{{\left( \sqrt{a}+\sqrt{c} \right)}^{2}}-1 \right]\ge 0. \\
\end{align}\] Thật vậy, vì $a\ge b$ và $c \ge 0$, nên ta có \[2{{\left( \sqrt{a}+\sqrt{c} \right)}^{2}}=2\left( a+c+2\sqrt{ac} \right)=\left( a+c+a \right)+\left( c+4\sqrt{ac} \right)\ge a+c+b=1.\] Bài toán được giải quyết trọn vẹn ;).
Đẳng thức xảy ra khi và chỉ khi $a=b=c=\dfrac{1}{3}$ hoặc $a=1, \ b=c=0$ hoặc các hoán vị. $\blacksquare$


#332493 Topic bất đẳng thức THCS (2)

Gửi bởi LilTee trong 06-07-2012 - 11:47

Mọi người làm hăng quá, nháy mắt đã xong hết rùi, chứng tỏ VMF chúng ta rất nhiều nhân tài :) Post vài bài nữa cho cả nhà cùng làm. Topic sôi nổi lên nào... Tra....zố.... :ukliam2:
Bài 416: Cho các số thực không âm $a,b,c$ thoả mãn $a+b+c=4$.CMR:

$a^{2}b+b^{2}c+c^{2}a+abc\geq 4$

Vasile Cirtoaje, Phạm Kim Hùng

Bài 417: Cho các số thực không âm $a,b,c$ thoả mãn không có hai số nào đòng thời bằng 0. Chứng minh rằng:

$\frac{\sum a^{2}}{\sum ab}+\frac{4abc}{a^{2}b+b^{2}c+c^{2}a+abc}\geq 2$.

Võ Quốc Bá Cẩn

Bài 418: Cho các số thực không âm $a,b,c$ thoả mãn không có 2 số nào đồng thời bằng không và $a^{2}+b^{2}+c^{2}=2(ab+bc+ca)$. Chứng minh rằng:

$\sqrt{\frac{ab}{a^{2}+b^{2}}}++\sqrt{\frac{bc}{b^{2}+c^{2}}}+\sqrt{\frac{ca}{c^{2}+a^{2}}}\geq \frac{1}{\sqrt{2}}$.

Bài 416: Cho $a=0,b=1,c=3$ bất đẳng thức sai ngay.
Bài 417: Giả sử $b$ là số nằm giữa, khi đó ta có ${{a}^{2}}b+{{b}^{2}}c+{{c}^{2}}a+abc\le b{{(a+c)}^{2}}.$
Như vậy, $$\begin{align}
\frac{\sum{{{a}^{2}}}}{\sum{a}b}+\frac{4abc}{{{a}^{2}}b+{{b}^{2}}c+{{c}^{2}}a+abc} & \ge \frac{\sum{{{a}^{2}}}}{\sum{a}b}+\frac{4ac}{{{\left( a+c \right)}^{2}}} \\
& =2+\frac{{{\left( {{a}^{2}}+{{c}^{2}}-ab-bc \right)}^{2}}}{{{\left( a+c \right)}^{2}}\sum{a}b} \\
& \ge 2. \\
\end{align}$$
Bài toán được chứng minh xong ;).
Đẳng thức xảy ra khi một biến bằng không, hai biến còn lại bằng nhau hoặc cả ba biến bằng nhau.
Bài 418 Theo $AM-GM$, ta có
$$\sum\limits_{cyc}{\sqrt{\frac{ab}{{{a}^{2}}+{{b}^{2}}}}}=\sum\limits_{cyc}{\frac{\sqrt{ab\left( {{a}^{2}}+{{b}^{2}} \right)}}{{{a}^{2}}+{{b}^{2}}}}\ge \sum\limits_{cyc}{\frac{\sqrt{2}ab}{{{a}^{2}}+{{b}^{2}}}}\ge \sum\limits_{cyc}{\frac{\sqrt{2}ab}{{{a}^{2}}+{{b}^{2}}+{{c}^{2}}}}=\sqrt{2}.\frac{ab+bc+ca}{{{a}^{2}}+{{b}^{2}}+{{c}^{2}}}=\frac{\sqrt{2}}{2}.$$
Bài toán được chứng minh xong ;).
Đẳng thức xảy ra khi một biến bằng không, hai biến còn lại bằng nhau. $\blacksquare$


#332483 [TS ĐH 2012] Đề thi và đáp án môn Toán khối A, A1

Gửi bởi LilTee trong 06-07-2012 - 11:13

A Kiên ơi e nghĩ hướng bđt này không cần phải Cô si 3 số đầu ch0 dài đâu mà đến chỗ:
$P\geq 3^{|x-y|}+3^{|y-z|}+3^{|z-x|}-(|x-y|+|y-z|+|z-x|)$
Chỉ cần chứng minh $f(t)=3^t-t-1\geq 0$ với mọi $t\geq 0$
$\to f'(t)=3^t.ln3-1\geq 0$ Nên $f(t)=3^t-t-1\geq f(0)=0$
Áp dụng với $t=|x-y|,|y-z|,|z-x|$ rồi cộng lại là có $P\geq 3$

Dám chém anh là không biết đạo hàm. Thế mà đạo hàm ầm ầm ấy ;)). Giết giờ :P
___
=)) Thông minh đột xuất anh à.


#308719 $$a^{-a}+b^{-b} \le 2$$

Gửi bởi LilTee trong 07-04-2012 - 10:55

Mở rộng bài toán :
Cho các số dương $a, b, c$ thỏa mãn :$abc = 1$. Chứng minh rằng :
$$a^{-a} + b^{-b} + c^{-c} \le 3$$

Ta đưa về chứng minh bất đẳng thức mạnh hơn là \[\frac{1}{{a}^{2}-a+1}+\frac{1}{{b}^{2}-b+1}+\frac{1}{{c}^{2}-c+1}\leq 3.\] Thật vậy, $$\sum\limits_{cyc}{\frac{1}{{{a}^{2}}-a+1}}=\sum\limits_{cyc}{\frac{3({{a}^{2}}+1)}{2({{a}^{4}}+{{a}^{2}}+1)}}-\sum\limits_{cyc}{\frac{{{\left( a-1 \right)}^{2}}}{2\left[ {{\left( {{a}^{2}}+1 \right)}^{2}}-{{a}^{2}} \right]}}\le \sum\limits_{cyc}{\frac{3({{a}^{2}}+1)}{2({{a}^{4}}+{{a}^{2}}+1)}}\overset{Vasc}{\mathop \le }\,3.$$ Bài toán được chứng minh hoàn toàn ;).
Đẳng thức xảy ra khi $a=b=c=1 \ \blacksquare$


#308713 Topic về bất đẳng thức

Gửi bởi LilTee trong 07-04-2012 - 09:55

Bài 72, Cho a,b,c dương t/m abc =1.CMR
\[\frac{{a(3a + 1)}}{{{{(a + 1)}^2}}} + \frac{{b(3b + 1)}}{{{{(b + 1)}^2}}} + \frac{{c(3c + 1)}}{{{{(c + 1)}^2}}} \ge 3\]

Đổi biến $\left( \frac{1}{a},\frac{1}{b},\frac{1}{c} \right)=\left( {{x}^{4}},{{y}^{4}},{{z}^{4}} \right)$. Ta cần chứng minh $$\sum\limits_{cyc}{\frac{{{x}^{4}}+3}{{{\left( {{x}^{4}}+1 \right)}^{2}}}}\ge 3.$$ Thậy vậy, theo bất đẳng thức quen thuộc của $Vasc$, ta có $$\sum\limits_{cyc}{\frac{{{x}^{4}}+3}{{{\left( {{x}^{4}}+1 \right)}^{2}}}}=3\sum\limits_{cyc}{\frac{1}{{{x}^{6}}+{{x}^{3}}+1}}+\sum\limits_{cyc}{\frac{{{\left( x-1 \right)}^{2}}{{x}^{3}}\left( {{x}^{5}}+2{{x}^{4}}-{{x}^{2}}+x+3 \right)}{{{\left( {{x}^{4}}+1 \right)}^{2}}\left( {{x}^{6}}+{{x}^{3}}+1 \right)}}\ge 3.$$ Bài toán được chứng minh xong ;).
Đẳng thức xảy ra khi $a=b=c=1. \ \blacksquare$


Bài 72, Cho a,b,c dương t/m abc =1.CMR
\[\frac{{{{(a - 3)}^2}}}{{{a^2} + 2}} + \frac{{{{(b - 3)}^2}}}{{{b^2} + 2}} + \frac{{{{(c - 3)}^2}}}{{{c^2} + 2}} \ge 4\]

Tạm thời chưa nghĩ ra cách nào khác, ngoài cách này. Tuy nhiên việc chứng minh tính đúng của bất đẳng thức sau là rất khó !
Chúng ta luôn có $$\frac{{{(a-3)}^{2}}}{{{a}^{2}}+2}\ge \frac{4}{{{a}^{2}}+a+1}-\frac{8}{9}\ln a.$$
Thiết lập hai biểu thức tương tự, sau đó sử dụng bất đẳng thức của $Vasc$ là ta có điều phải chứng minh.
Đẳng thức xảy ra khi $a=b=c=1. \ \blacksquare$